Bounded, measurable and supported on a set of finite measure function












2












$begingroup$


Suppose f is a bounded and measurable function on R and supported on a set of finite measure. Prove that for every $epsilon gt 0$ there exists a simple function $s$ such that $int |f-s|dx$ $lt epsilon$



I have question: Can I let this simple function be supported on a set of finite measure? Then this simple function will be bounded and measurable and supported, then I can use the linearity of integral and the definition of the integral of this kind of functions to prove that.



I am wondering if I am right. I really appreciate your help if you would like to give me another proof.



Thanks










share|cite|improve this question









$endgroup$

















    2












    $begingroup$


    Suppose f is a bounded and measurable function on R and supported on a set of finite measure. Prove that for every $epsilon gt 0$ there exists a simple function $s$ such that $int |f-s|dx$ $lt epsilon$



    I have question: Can I let this simple function be supported on a set of finite measure? Then this simple function will be bounded and measurable and supported, then I can use the linearity of integral and the definition of the integral of this kind of functions to prove that.



    I am wondering if I am right. I really appreciate your help if you would like to give me another proof.



    Thanks










    share|cite|improve this question









    $endgroup$















      2












      2








      2





      $begingroup$


      Suppose f is a bounded and measurable function on R and supported on a set of finite measure. Prove that for every $epsilon gt 0$ there exists a simple function $s$ such that $int |f-s|dx$ $lt epsilon$



      I have question: Can I let this simple function be supported on a set of finite measure? Then this simple function will be bounded and measurable and supported, then I can use the linearity of integral and the definition of the integral of this kind of functions to prove that.



      I am wondering if I am right. I really appreciate your help if you would like to give me another proof.



      Thanks










      share|cite|improve this question









      $endgroup$




      Suppose f is a bounded and measurable function on R and supported on a set of finite measure. Prove that for every $epsilon gt 0$ there exists a simple function $s$ such that $int |f-s|dx$ $lt epsilon$



      I have question: Can I let this simple function be supported on a set of finite measure? Then this simple function will be bounded and measurable and supported, then I can use the linearity of integral and the definition of the integral of this kind of functions to prove that.



      I am wondering if I am right. I really appreciate your help if you would like to give me another proof.



      Thanks







      real-analysis






      share|cite|improve this question













      share|cite|improve this question











      share|cite|improve this question




      share|cite|improve this question










      asked Dec 8 '13 at 17:28









      YangYang

      206417




      206417






















          3 Answers
          3






          active

          oldest

          votes


















          0












          $begingroup$

          You can choose any simple function that satisfies the condition whether it is supported by finite set or not. In fact, simple function is always bounded and measurable without any condition.






          share|cite|improve this answer









          $endgroup$













          • $begingroup$
            Can you give me another proof of this question, Wit@Worawit Tepsan
            $endgroup$
            – Yang
            Dec 8 '13 at 20:10



















          0












          $begingroup$

          We assume that $f$ is supported on $E$ which $m(E)=L<infty.$ So we can consider a function $f$ on E.
          Since $f$ is bounded, it must have a and b such that
          $a leq f(x) leq b$ for all $xin E$. Let $N > frac{L(b-a)}{varepsilon}.$



          We define $E_i = {x in E : a+i(b-a)/N leq f(x) < a+(i+1)(b-a)/N}$.



          Choose $x_iin E_i$ and define

          $ s(x) = sum_{i=0}^{N} f(x_i) lambda(E_i)$.



          Then you can show that



          $|f(x)-s(x)|leq frac{varepsilon}{L}.$



          And so on.






          share|cite|improve this answer











          $endgroup$













          • $begingroup$
            the question is that the domain is R which has infinity measure@Worawit Tepsan
            $endgroup$
            – Yang
            Dec 8 '13 at 21:14










          • $begingroup$
            I edited the proof.
            $endgroup$
            – Worawit Tepsan
            Dec 8 '13 at 21:41








          • 1




            $begingroup$
            @Worawit Tepsan, can you please elaborate on how you can show that $|f(x) - s(x)|leq frac{epsilon}{L}$?
            $endgroup$
            – ALannister
            Nov 18 '15 at 15:17










          • $begingroup$
            I did typo. I should choose $N>frac{L(b-a)}{varepsilon}.$
            $endgroup$
            – Worawit Tepsan
            Nov 29 '15 at 3:33





















          0












          $begingroup$

          Let $X$ be the support of $f$ and $mathbb{1}_X$ its characteristic function. Now observe that $f=fcdot mathbb{1}_X$ and that $s cdot mathbb{1}_X$ approximate $f$.






          share|cite|improve this answer









          $endgroup$













            Your Answer





            StackExchange.ifUsing("editor", function () {
            return StackExchange.using("mathjaxEditing", function () {
            StackExchange.MarkdownEditor.creationCallbacks.add(function (editor, postfix) {
            StackExchange.mathjaxEditing.prepareWmdForMathJax(editor, postfix, [["$", "$"], ["\\(","\\)"]]);
            });
            });
            }, "mathjax-editing");

            StackExchange.ready(function() {
            var channelOptions = {
            tags: "".split(" "),
            id: "69"
            };
            initTagRenderer("".split(" "), "".split(" "), channelOptions);

            StackExchange.using("externalEditor", function() {
            // Have to fire editor after snippets, if snippets enabled
            if (StackExchange.settings.snippets.snippetsEnabled) {
            StackExchange.using("snippets", function() {
            createEditor();
            });
            }
            else {
            createEditor();
            }
            });

            function createEditor() {
            StackExchange.prepareEditor({
            heartbeatType: 'answer',
            autoActivateHeartbeat: false,
            convertImagesToLinks: true,
            noModals: true,
            showLowRepImageUploadWarning: true,
            reputationToPostImages: 10,
            bindNavPrevention: true,
            postfix: "",
            imageUploader: {
            brandingHtml: "Powered by u003ca class="icon-imgur-white" href="https://imgur.com/"u003eu003c/au003e",
            contentPolicyHtml: "User contributions licensed under u003ca href="https://creativecommons.org/licenses/by-sa/3.0/"u003ecc by-sa 3.0 with attribution requiredu003c/au003e u003ca href="https://stackoverflow.com/legal/content-policy"u003e(content policy)u003c/au003e",
            allowUrls: true
            },
            noCode: true, onDemand: true,
            discardSelector: ".discard-answer"
            ,immediatelyShowMarkdownHelp:true
            });


            }
            });














            draft saved

            draft discarded


















            StackExchange.ready(
            function () {
            StackExchange.openid.initPostLogin('.new-post-login', 'https%3a%2f%2fmath.stackexchange.com%2fquestions%2f598402%2fbounded-measurable-and-supported-on-a-set-of-finite-measure-function%23new-answer', 'question_page');
            }
            );

            Post as a guest















            Required, but never shown

























            3 Answers
            3






            active

            oldest

            votes








            3 Answers
            3






            active

            oldest

            votes









            active

            oldest

            votes






            active

            oldest

            votes









            0












            $begingroup$

            You can choose any simple function that satisfies the condition whether it is supported by finite set or not. In fact, simple function is always bounded and measurable without any condition.






            share|cite|improve this answer









            $endgroup$













            • $begingroup$
              Can you give me another proof of this question, Wit@Worawit Tepsan
              $endgroup$
              – Yang
              Dec 8 '13 at 20:10
















            0












            $begingroup$

            You can choose any simple function that satisfies the condition whether it is supported by finite set or not. In fact, simple function is always bounded and measurable without any condition.






            share|cite|improve this answer









            $endgroup$













            • $begingroup$
              Can you give me another proof of this question, Wit@Worawit Tepsan
              $endgroup$
              – Yang
              Dec 8 '13 at 20:10














            0












            0








            0





            $begingroup$

            You can choose any simple function that satisfies the condition whether it is supported by finite set or not. In fact, simple function is always bounded and measurable without any condition.






            share|cite|improve this answer









            $endgroup$



            You can choose any simple function that satisfies the condition whether it is supported by finite set or not. In fact, simple function is always bounded and measurable without any condition.







            share|cite|improve this answer












            share|cite|improve this answer



            share|cite|improve this answer










            answered Dec 8 '13 at 18:13









            Worawit TepsanWorawit Tepsan

            333212




            333212












            • $begingroup$
              Can you give me another proof of this question, Wit@Worawit Tepsan
              $endgroup$
              – Yang
              Dec 8 '13 at 20:10


















            • $begingroup$
              Can you give me another proof of this question, Wit@Worawit Tepsan
              $endgroup$
              – Yang
              Dec 8 '13 at 20:10
















            $begingroup$
            Can you give me another proof of this question, Wit@Worawit Tepsan
            $endgroup$
            – Yang
            Dec 8 '13 at 20:10




            $begingroup$
            Can you give me another proof of this question, Wit@Worawit Tepsan
            $endgroup$
            – Yang
            Dec 8 '13 at 20:10











            0












            $begingroup$

            We assume that $f$ is supported on $E$ which $m(E)=L<infty.$ So we can consider a function $f$ on E.
            Since $f$ is bounded, it must have a and b such that
            $a leq f(x) leq b$ for all $xin E$. Let $N > frac{L(b-a)}{varepsilon}.$



            We define $E_i = {x in E : a+i(b-a)/N leq f(x) < a+(i+1)(b-a)/N}$.



            Choose $x_iin E_i$ and define

            $ s(x) = sum_{i=0}^{N} f(x_i) lambda(E_i)$.



            Then you can show that



            $|f(x)-s(x)|leq frac{varepsilon}{L}.$



            And so on.






            share|cite|improve this answer











            $endgroup$













            • $begingroup$
              the question is that the domain is R which has infinity measure@Worawit Tepsan
              $endgroup$
              – Yang
              Dec 8 '13 at 21:14










            • $begingroup$
              I edited the proof.
              $endgroup$
              – Worawit Tepsan
              Dec 8 '13 at 21:41








            • 1




              $begingroup$
              @Worawit Tepsan, can you please elaborate on how you can show that $|f(x) - s(x)|leq frac{epsilon}{L}$?
              $endgroup$
              – ALannister
              Nov 18 '15 at 15:17










            • $begingroup$
              I did typo. I should choose $N>frac{L(b-a)}{varepsilon}.$
              $endgroup$
              – Worawit Tepsan
              Nov 29 '15 at 3:33


















            0












            $begingroup$

            We assume that $f$ is supported on $E$ which $m(E)=L<infty.$ So we can consider a function $f$ on E.
            Since $f$ is bounded, it must have a and b such that
            $a leq f(x) leq b$ for all $xin E$. Let $N > frac{L(b-a)}{varepsilon}.$



            We define $E_i = {x in E : a+i(b-a)/N leq f(x) < a+(i+1)(b-a)/N}$.



            Choose $x_iin E_i$ and define

            $ s(x) = sum_{i=0}^{N} f(x_i) lambda(E_i)$.



            Then you can show that



            $|f(x)-s(x)|leq frac{varepsilon}{L}.$



            And so on.






            share|cite|improve this answer











            $endgroup$













            • $begingroup$
              the question is that the domain is R which has infinity measure@Worawit Tepsan
              $endgroup$
              – Yang
              Dec 8 '13 at 21:14










            • $begingroup$
              I edited the proof.
              $endgroup$
              – Worawit Tepsan
              Dec 8 '13 at 21:41








            • 1




              $begingroup$
              @Worawit Tepsan, can you please elaborate on how you can show that $|f(x) - s(x)|leq frac{epsilon}{L}$?
              $endgroup$
              – ALannister
              Nov 18 '15 at 15:17










            • $begingroup$
              I did typo. I should choose $N>frac{L(b-a)}{varepsilon}.$
              $endgroup$
              – Worawit Tepsan
              Nov 29 '15 at 3:33
















            0












            0








            0





            $begingroup$

            We assume that $f$ is supported on $E$ which $m(E)=L<infty.$ So we can consider a function $f$ on E.
            Since $f$ is bounded, it must have a and b such that
            $a leq f(x) leq b$ for all $xin E$. Let $N > frac{L(b-a)}{varepsilon}.$



            We define $E_i = {x in E : a+i(b-a)/N leq f(x) < a+(i+1)(b-a)/N}$.



            Choose $x_iin E_i$ and define

            $ s(x) = sum_{i=0}^{N} f(x_i) lambda(E_i)$.



            Then you can show that



            $|f(x)-s(x)|leq frac{varepsilon}{L}.$



            And so on.






            share|cite|improve this answer











            $endgroup$



            We assume that $f$ is supported on $E$ which $m(E)=L<infty.$ So we can consider a function $f$ on E.
            Since $f$ is bounded, it must have a and b such that
            $a leq f(x) leq b$ for all $xin E$. Let $N > frac{L(b-a)}{varepsilon}.$



            We define $E_i = {x in E : a+i(b-a)/N leq f(x) < a+(i+1)(b-a)/N}$.



            Choose $x_iin E_i$ and define

            $ s(x) = sum_{i=0}^{N} f(x_i) lambda(E_i)$.



            Then you can show that



            $|f(x)-s(x)|leq frac{varepsilon}{L}.$



            And so on.







            share|cite|improve this answer














            share|cite|improve this answer



            share|cite|improve this answer








            edited Nov 29 '15 at 3:34

























            answered Dec 8 '13 at 20:49









            Worawit TepsanWorawit Tepsan

            333212




            333212












            • $begingroup$
              the question is that the domain is R which has infinity measure@Worawit Tepsan
              $endgroup$
              – Yang
              Dec 8 '13 at 21:14










            • $begingroup$
              I edited the proof.
              $endgroup$
              – Worawit Tepsan
              Dec 8 '13 at 21:41








            • 1




              $begingroup$
              @Worawit Tepsan, can you please elaborate on how you can show that $|f(x) - s(x)|leq frac{epsilon}{L}$?
              $endgroup$
              – ALannister
              Nov 18 '15 at 15:17










            • $begingroup$
              I did typo. I should choose $N>frac{L(b-a)}{varepsilon}.$
              $endgroup$
              – Worawit Tepsan
              Nov 29 '15 at 3:33




















            • $begingroup$
              the question is that the domain is R which has infinity measure@Worawit Tepsan
              $endgroup$
              – Yang
              Dec 8 '13 at 21:14










            • $begingroup$
              I edited the proof.
              $endgroup$
              – Worawit Tepsan
              Dec 8 '13 at 21:41








            • 1




              $begingroup$
              @Worawit Tepsan, can you please elaborate on how you can show that $|f(x) - s(x)|leq frac{epsilon}{L}$?
              $endgroup$
              – ALannister
              Nov 18 '15 at 15:17










            • $begingroup$
              I did typo. I should choose $N>frac{L(b-a)}{varepsilon}.$
              $endgroup$
              – Worawit Tepsan
              Nov 29 '15 at 3:33


















            $begingroup$
            the question is that the domain is R which has infinity measure@Worawit Tepsan
            $endgroup$
            – Yang
            Dec 8 '13 at 21:14




            $begingroup$
            the question is that the domain is R which has infinity measure@Worawit Tepsan
            $endgroup$
            – Yang
            Dec 8 '13 at 21:14












            $begingroup$
            I edited the proof.
            $endgroup$
            – Worawit Tepsan
            Dec 8 '13 at 21:41






            $begingroup$
            I edited the proof.
            $endgroup$
            – Worawit Tepsan
            Dec 8 '13 at 21:41






            1




            1




            $begingroup$
            @Worawit Tepsan, can you please elaborate on how you can show that $|f(x) - s(x)|leq frac{epsilon}{L}$?
            $endgroup$
            – ALannister
            Nov 18 '15 at 15:17




            $begingroup$
            @Worawit Tepsan, can you please elaborate on how you can show that $|f(x) - s(x)|leq frac{epsilon}{L}$?
            $endgroup$
            – ALannister
            Nov 18 '15 at 15:17












            $begingroup$
            I did typo. I should choose $N>frac{L(b-a)}{varepsilon}.$
            $endgroup$
            – Worawit Tepsan
            Nov 29 '15 at 3:33






            $begingroup$
            I did typo. I should choose $N>frac{L(b-a)}{varepsilon}.$
            $endgroup$
            – Worawit Tepsan
            Nov 29 '15 at 3:33













            0












            $begingroup$

            Let $X$ be the support of $f$ and $mathbb{1}_X$ its characteristic function. Now observe that $f=fcdot mathbb{1}_X$ and that $s cdot mathbb{1}_X$ approximate $f$.






            share|cite|improve this answer









            $endgroup$


















              0












              $begingroup$

              Let $X$ be the support of $f$ and $mathbb{1}_X$ its characteristic function. Now observe that $f=fcdot mathbb{1}_X$ and that $s cdot mathbb{1}_X$ approximate $f$.






              share|cite|improve this answer









              $endgroup$
















                0












                0








                0





                $begingroup$

                Let $X$ be the support of $f$ and $mathbb{1}_X$ its characteristic function. Now observe that $f=fcdot mathbb{1}_X$ and that $s cdot mathbb{1}_X$ approximate $f$.






                share|cite|improve this answer









                $endgroup$



                Let $X$ be the support of $f$ and $mathbb{1}_X$ its characteristic function. Now observe that $f=fcdot mathbb{1}_X$ and that $s cdot mathbb{1}_X$ approximate $f$.







                share|cite|improve this answer












                share|cite|improve this answer



                share|cite|improve this answer










                answered Jan 19 at 12:20









                lzralbulzralbu

                640512




                640512






























                    draft saved

                    draft discarded




















































                    Thanks for contributing an answer to Mathematics Stack Exchange!


                    • Please be sure to answer the question. Provide details and share your research!

                    But avoid



                    • Asking for help, clarification, or responding to other answers.

                    • Making statements based on opinion; back them up with references or personal experience.


                    Use MathJax to format equations. MathJax reference.


                    To learn more, see our tips on writing great answers.




                    draft saved


                    draft discarded














                    StackExchange.ready(
                    function () {
                    StackExchange.openid.initPostLogin('.new-post-login', 'https%3a%2f%2fmath.stackexchange.com%2fquestions%2f598402%2fbounded-measurable-and-supported-on-a-set-of-finite-measure-function%23new-answer', 'question_page');
                    }
                    );

                    Post as a guest















                    Required, but never shown





















































                    Required, but never shown














                    Required, but never shown












                    Required, but never shown







                    Required, but never shown

































                    Required, but never shown














                    Required, but never shown












                    Required, but never shown







                    Required, but never shown







                    Popular posts from this blog

                    Can a sorcerer learn a 5th-level spell early by creating spell slots using the Font of Magic feature?

                    Does disintegrating a polymorphed enemy still kill it after the 2018 errata?

                    A Topological Invariant for $pi_3(U(n))$